Lorentz Boosts in the same direction should form a group. Two boosts along the x axis should produce another boost along the x axis. Is that correct?

Hagman7v

Hagman7v

Answered question

2022-09-26

Lorentz Boosts in the same direction should form a group. Two boosts along the x axis should produce another boost along the x axis. Is that correct?

Answer & Explanation

vyhlodatis

vyhlodatis

Beginner2022-09-27Added 14 answers

Yes, it is a one-dimensional subgroup generated by exponentiating an infinitesimal boost. Every one dimensional exponentiation of a generator forms an abelian group, because e a G e b G = e ( a + b ) G there is nothing to not commute. This result is the addition of velocities, you can explicity check that this is associative (it is always manifestly commutative).

Do you have a similar question?

Recalculate according to your conditions!

New Questions in Relativity

Ask your question.
Get an expert answer.

Let our experts help you. Answer in as fast as 15 minutes.

Didn't find what you were looking for?